Đến nội dung

Hình ảnh

Topic tổng hợp một số bất đẳng thức trong kì thi MO các nước

* * * * * 16 Bình chọn

  • Please log in to reply
Chủ đề này có 501 trả lời

#221
dogsteven

dogsteven

    Đại úy

  • Thành viên
  • 1567 Bài viết

 

Bài 62:
Trường hợp 1:$min{\{a,b,c}\} \geq -\frac{3}{4} \Rightarrow a,b,c \geq -\frac{3}{4}$
Ta cần chứng minh $\frac{a}{a^2+1} \leq \frac{36a+3}{50}$ (do xài pp tiếp tuyến nên biết cần cm cái này)
$ \Leftrightarrow \frac{(4a+3)(3a-1)^2}{50(a^2+1)} \geq 0 $
Tương tự ta được: $ \sum\frac{a}{a^2+1} \leq \frac{9}{10}$
Trường hợp 2:$min{\{a,b,c}\} < -\frac{3}{4} $
Giả sử $ c=min{\{a,b,c}\} \Rightarrow c<\frac{3}{4} $
Nếu $ a \geq 0 , b \leq 0 $ thì  $\frac{b}{b^2+1}+\frac{c}{c^2+1} <0$ và $\frac{a}{a^2+1} \leq \frac{1}{2}$ nên $VT < \frac{1}{2} < \frac{9}{10}$
Tương tự với $a \leq 0;b \geq 0$
Nếu $a \leq 0;b \leq 0$ thì $VT<0 < \frac{9}{10} $
Nếu $a \geq 0;b \geq 0$ thì ta có:

  1. Nếu $a \in [0;\frac{1}{2}] \cup [2;+\infty)$ thì $ \frac{a}{a^2+1} \leq \frac{2}{5} $.Kết hợp với $\frac{b}{b^2+1} \leq \frac{1}{2} ; \frac{c}{c^2+1} <0$ ta được $VT < \frac{9}{10}$
  2. Nếu $a \in [\frac{1}{2};2] $.
    -Xét $b \in [0;\frac{1}{2}] \cup [2;+\infty)$ thì  $ \frac{b}{b^2+1} \leq \frac{2}{5} $
    Kết hợp với $\frac{a}{a^2+1} \leq \frac{1}{2} ; \frac{c}{c^2+1} <0$ ta được $VT < \frac{9}{10}$
    -Xét $b \in [\frac{1}{2};2]$ thì ta có:$c=1-a-b \geq -3$
    Do $c \in [-3;-\frac{3}{4})$ nên $\frac{c}{c^2+1} < -\frac{1}{10}$
    Do đó ta có:$VT<\frac{1}{2}+\frac{1}{2}-\frac{1}{10}=\frac{9}{10} $.
    Vậy $\sum\frac{a}{a^2+1} \leq \frac{9}{10} $.Dấu "=" xảy ra khi $a=b=c=\frac{1}{3}$

 

Bất đẳng thức tương đương với: $\sum \dfrac{(a-1)^2}{a^2+1}\geqslant \dfrac{6}{5}$

Giả sử $(3b-1)(3c-1)\geqslant 0\Leftrightarrow b^2+c^2\leqslant \dfrac{1}{9}+\left(\dfrac{2}{3}-a\right)^2$

Đến đây ta có $\dfrac{(b-1)^2}{b^2+1}+\dfrac{(c-1)^2}{c^2+1}\geqslant \dfrac{9(a+1)^2}{19+\left(2-3a\right)^2}$

Đến đây biến đổi tương đương.


Quyết tâm off dài dài cày hình, số, tổ, rời rạc.


#222
Namthemaster1234

Namthemaster1234

    Thiếu úy

  • Thành viên
  • 550 Bài viết

 Bài 90: (MOSP 2003) Cho 3 số thực không âm a,b,c thỏa mãn $a^2+b^2+c^2=1$ . Chứng minh rằng:

 

 $1\leq \sum \frac{a}{1+bc}\leq \frac{3\sqrt{3}}{4}$


Đừng lo lắng về khó khăn của bạn trong toán học, tôi đảm bảo với bạn rằng những khó khăn toán học của tôi còn gấp bội.
(Albert Einstein)

Visit my facebook: https://www.facebook.com/cao.simon.56

:icon6: :icon6: :icon6: :icon6: :icon6:


#223
Belphegor Varia

Belphegor Varia

    Thượng sĩ

  • Thành viên
  • 227 Bài viết

Bài 91 (VMO 1998A) : Tìm GTNN của biểu thức :
$F(x,y)=\sqrt{(x+1)^{2}+(y-1)^{2}}+\sqrt{(x-1)^{2}+(y+1)^{2}}+\sqrt{(x+2)^{2}+(y+2)^{2}}$
 

Bài 92 (VMO 1998B) : Cho $n$ số thực dương $x_{1},x_{2},...,x_{n} (n\geq 2)$ thỏa mãn 
$\frac{1}{x_{1}+1998}+\frac{1}{x_{2}+1998}+...+\frac{1}{x_{n}+1998}=\frac{1}{1998}$

 

Chứng minh rằng : $\frac{\sqrt[n]{x_{1}x_{2}...x_{n}}}{n-1}\geq 1998$
 


Bài viết đã được chỉnh sửa nội dung bởi khanghaxuan: 09-06-2015 - 09:51

$ \textbf{NMQ}$

Wait a minute, You have enough time. Also tomorrow will come 

Just take off her or give me a ride 

Give me one day or one hour or just one minute for a short word 

 


#224
khanghaxuan

khanghaxuan

    Trung úy

  • Thành viên
  • 969 Bài viết

Lời giải bài 92 : Cho $n$ số thực dương $x_{1},x_{2},...,x_{n} (n\geq 2)$ thỏa mãn 

$\frac{1}{x_{1}+1998}+\frac{1}{x_{2}+1998}+...+\frac{1}{x_{n}+1998}=\frac{1}{1998}$

Chứng minh rằng : $\frac{\sqrt[n]{x_{1}x_{2}...x_{n}}}{n-1}\geq 1998$

 

Thoạt đầu , nhìn sơ qua bài toán trên mình có cảm giác như nó gần giống với bài toán quen  thuộc sau : 

Cho $a,b,c,d>0$ thỏa mãn : $\sum \frac{1}{a^{4}+1}=1$ . Chứng minh rằng : $abcd\geq 3$

Bài toán nhỏ trên quen thuộc với hầu hết với các mem . Do đó , một cách thình lình ta nghĩ tới việc giải bài toán lớn trên bằng pp của bài toán con đó . 

Ta có : $\sum_{i=1}^{1998}\frac{1}{x_{i}+1998}=\frac{1}{1998}\Rightarrow \sum_{i=2}^{1998}\frac{1}{x_{i}+1998}=\frac{1}{1998}-\frac{1}{x_{1}+1998}=\frac{x_{1}}{1998(x_{1}+1998)}\Rightarrow \frac{x_{1}}{1998(x_{1}+1998)}\geq 1997\sqrt[1997]{\frac{1}{\prod_{i=2}^{1998}(x_{i}+1998)}}$

Lập các bđt tương tự rồi nhân vế theo vế ta được  

$\frac{\prod_{i=1}^{1998}(x_{i})}{1998^{n}}\geq (n-1)^{n}\Rightarrow \sqrt[n]{\prod_{i=1}^{1998}(x_{i})}\geq 1998.(n-1)$ (ĐPCM :)) )


Điều tôi muốn biết trước tiên không phải là bạn đã thất bại ra sao mà là bạn đã chấp nhận nó như thế nào .

- A.Lincoln -

#225
Belphegor Varia

Belphegor Varia

    Thượng sĩ

  • Thành viên
  • 227 Bài viết

Mạn phép làm thử cách 2 Bài 92 như này :D

Ta có đẳng thức đề bài tương đương với : $\frac{1998}{x_{1}+1998}+\frac{1998}{x_{2}+1998}+...+\frac{1998}{x_{n}+1998}=1$
Đặt $y_{i}=\frac{x_{i}}{1998}$ với $i=1,2,...,n$ thì đẳng thức trên trở thành :
$\frac{1}{y_{1}+1}+\frac{1}{y_{2}+1}+...+\frac{1}{y_{n}+1}= 1$
Ta cần chứng minh $\sqrt[n]{x_{1}x_{2}...x_{n}}\geq 1998(n-1)\Leftrightarrow x_{1}x_{2}...x_{n}\geq 1998^{n}(n-1)^{n}$ hay $y_{1}y_{2}...y_{n}\geq (n-1)^{n}$
Đặt $a_{i}=\frac{1}{y_{i}+1}$ $(i=1,2,...,n)$ 
Khi đó ta có $\sum_{i=1}^{n}a_{i}=1$ . Đặt $A=\prod_{i=1}^{n}a_{i}$ .
Sử dụng BĐT $AM-GM$ ta có :
$1-a_{i}=\sum_{j=1(j\neq i)}^{n}a_{j}\geq (n-1)\sqrt[n-1]{\prod_{j=1(j\neq i)}^{n}a_{j}}$ $=(n-1)\sqrt[n-1]{\frac{A}{a_{i}}}$
Khi đó $\prod_{i=1}^{n}y_{i}=\prod_{i=1}^{n}\frac{1-a_{1}}{ai}=\frac{1}{\prod_{i=1}^{n}ai}.\prod_{i=1}^{n}(1-a_{i})\geq \frac{1}{A}.(n-1)^{n}.\prod_{i=1}^{n}\sqrt[n-1]{\frac{A}{a_{i}}}=\frac{1}{A}(n-1)^{n}.\frac{\sqrt[n-1]{A^{n}}}{\prod_{i=1}^{n}\sqrt[n-1]{a_{i}}}=(n-1)^{n}.\frac{\sqrt[n-1]{A}}{\sqrt[n-1]{A}}=(n-1)^{n}$ 
 

 

PS : Gõ xong mệt muốn chết  :wacko:  :wacko:

 

@Khanghaxuan : Chú mày màu quá trời :P


Bài viết đã được chỉnh sửa nội dung bởi Belphegor Varia: 09-06-2015 - 14:59

$ \textbf{NMQ}$

Wait a minute, You have enough time. Also tomorrow will come 

Just take off her or give me a ride 

Give me one day or one hour or just one minute for a short word 

 


#226
khanghaxuan

khanghaxuan

    Trung úy

  • Thành viên
  • 969 Bài viết

Lời giải bài 91 : Tìm GTNN của biểu thức :

$F(x,y)=\sqrt{(x+1)^{2}+(y-1)^{2}}+\sqrt{(x-1)^{2}+(y+1)^{2}}+\sqrt{(x+2)^{2}+(y+2)^{2}}$

 

Đây là một bài toán được phát biểu dưới dạng toán quen thuộc . Ta sẽ biểu diễn trên hệ trục tọa độ $Oxy$ các điểm : 

$A(-1,1) ; B(-2,-2) ; C(1,-1)$

Tìm GTNN của $F(x,y)$ qua các biểu diễn trên là tìm GTNN của tổng khoảng cách từ điểm $M(x,y)$ đến ba điểm $A,B,C$

Từ đó bài toán quy về bài toán quen thuộc sau :) 

Cho tam giác ABC xác định và điểm M tùy ý . Tìm vị trí của M sao cho $MA+MB+MC$ nhỏ nhất

Mình sẽ làm đại diện : $M$ nằm trong tam giác . 

Điểm $M$ thỏa mãn điều kiện trên được  gọi là điểm torricelli trong tam giác và dấu $=$ xảy ra khi $\widehat{AMB}=\widehat{BMC}=\widehat{AMC}=120^{0}$

Do đó ta dễ dàng tìm được KQ của bài toán :)) 


Điều tôi muốn biết trước tiên không phải là bạn đã thất bại ra sao mà là bạn đã chấp nhận nó như thế nào .

- A.Lincoln -

#227
khanghaxuan

khanghaxuan

    Trung úy

  • Thành viên
  • 969 Bài viết

Bài 93 ( IMO 2012 ) : Cho $a_{2},a_{3},...,a_{n}$ thỏa mãn : $a_{2}.a_{3}...a_{n}=1$ . Chứng minh rằng : 

$(1+a_{2})^{2}(1+a_{3})^{3}...(1+a_{n})^{n}>n^{n}$


Điều tôi muốn biết trước tiên không phải là bạn đã thất bại ra sao mà là bạn đã chấp nhận nó như thế nào .

- A.Lincoln -

#228
Hoang Tung 126

Hoang Tung 126

    Thiếu tá

  • Thành viên
  • 2061 Bài viết

Bài 93 ( IMO 2012 ) : Cho $a_{2},a_{3},...,a_{n}$ thỏa mãn : $a_{2}.a_{3}...a_{n}=1$ . Chứng minh rằng : 

$(1+a_{2})^{2}(1+a_{3})^{3}...(1+a_{n})^{n}>n^{n}$

Theo Cosi ta có : 

 

  $1+a_{2}\geq 2\sqrt{a_{2}}= > (1+a_{2})^2\geq 2^2.a_{2}$

 $a_{3}+1=a_{3}+\frac{1}{2}+\frac{1}{2}\geq 3\sqrt[3]{\frac{a_{3}}{2^2}}= > (1+a_{3})^3\geq \frac{3^3.a_{3}}{2^2}$

  .....................................................................................................

 $(a_{n}+1)=a_{n}+\frac{1}{n-1}+\frac{1}{n-1}+...+\frac{1}{n-1}\geq n\sqrt[n]{\frac{a_{n}}{(n-1)^{n-1}}}= > (1+a_{n})^n\geq n^{n}.\frac{a_{n}}{(n-1)^{(n-1)}}$

 

 Nhân theo vế các bất đẳng thức 

 

 $= > (1+a_{2})^2(1+a_{3})^3...(1+a_{n})^n\geq 2^{2}.3^{3}...n^{n}.(a_{1}a_{2}...a_{n}).(\frac{1}{2^{2}}.\frac{1}{3^{3}}...\frac{1}{(n-1)^{n-1}})=n^{n}= > (1+a_{2})^2(1+a_{3})^3...(1+a_{n})^{n}\geq n^{n}$

      (Do $a_{2}a_{3}...a_{n}=1$)

 

 Dấu = xảy ra khi $a_{2}=1,a_{3}=\frac{1}{2},...a_{n}=\frac{1}{n-1}= > a_{2}a_{3}...a_{n}=\frac{1}{2.3...(n-1)}$

 

Nhưng điều này vô lý do $a_{2}a_{3}...a_{n}=1$

 

  Do đó dấu = ko xảy ra ,tức là $(a_{2}+1)^2(a_{3}+1)^3...(a_{n}+1)^{n}> n^{n}$



#229
khanghaxuan

khanghaxuan

    Trung úy

  • Thành viên
  • 969 Bài viết

Bài 94 : Cho $a_{1};a_{2};...;a_{2011}$ là các số thực không âm có tổng bằng : $\frac{2011}{2}$ . Chứng minh rằng: 

$\begin{vmatrix} (a_{1}-a_{2})(a_{2}-a_{3})...(a_{2011}-a_{1}) \end{vmatrix}\leq \frac{3\sqrt{3}}{16}$

Spoiler


Điều tôi muốn biết trước tiên không phải là bạn đã thất bại ra sao mà là bạn đã chấp nhận nó như thế nào .

- A.Lincoln -

#230
khanghaxuan

khanghaxuan

    Trung úy

  • Thành viên
  • 969 Bài viết

Bài 95 ( USAJMO 2011 ): Cho $a,b,c$ là các số thực dương thỏa mãn : $a^{2}+b^{2}+c^{2}+(a+b+c)^{2}\leq 4$ . Chứng minh rằng : 

$\frac{ab+1}{(a+b)^{2}}+\frac{bc+1}{(b+c)^{2}}+\frac{ca+1}{(c+a)^{2}}\geq 3$


Điều tôi muốn biết trước tiên không phải là bạn đã thất bại ra sao mà là bạn đã chấp nhận nó như thế nào .

- A.Lincoln -

#231
Hoang Tung 126

Hoang Tung 126

    Thiếu tá

  • Thành viên
  • 2061 Bài viết

Bài 95 ( USAJMO 2011 ): Cho $a,b,c$ là các số thực dương thỏa mãn : $a^{2}+b^{2}+c^{2}+(a+b+c)^{2}\leq 4$ . Chứng minh rằng : 

$\frac{ab+1}{(a+b)^{2}}+\frac{bc+1}{(b+c)^{2}}+\frac{ca+1}{(c+a)^{2}}\geq 3$

Ta có : $4\geq \sum a^2+(\sum a)^2=2(\sum a^2+\sum ab)= > 2\geq \sum a^2+\sum ab$

 

Từ đó :$\sum \frac{ab+1}{(a+b)^2}=\sum \frac{2ab+2}{2(a+b)^2}\geq \sum \frac{2ab+\sum a^2+\sum ab}{2(a+b)^2}=\sum \frac{(a+b)^2+(c+a)(c+b)}{2(a+b)^2}=\sum (\frac{1}{2}+\frac{(a+c)(c+b)}{2(a+b)^2})=\frac{3}{2}+\frac{1}{2}(\sum \frac{(c+a)(c+b)}{(a+b)^2})\geq \frac{3}{2}+\frac{1}{2}.3\sqrt[3]{\frac{(c+a)(c+b)}{(a+b)^2}.\frac{(b+c)(b+a)}{(c+a)^2}.\frac{(a+b)(a+c)}{(b+c)^2}}=\frac{3}{2}+\frac{3}{2}=3= > \sum \frac{ab+1}{(a+b)^2}\geq 3$

 

  Dấu = xảy ra khi $a=b=c=\frac{1}{\sqrt{3}}$ 



#232
Belphegor Varia

Belphegor Varia

    Thượng sĩ

  • Thành viên
  • 227 Bài viết

Bài 96 (VMO 2001A ): Xét các số thực dương $x,y,z$ thoả mãn hệ điều kiện sau :

$1$ , $\frac{1}{\sqrt{2}}\leq z<\frac{\min (x\sqrt{2},y\sqrt{3})}{2}$
$2$ , $x+z\sqrt{3}\geq \sqrt{6}$
$3$ ,  $y\sqrt{3}+z\sqrt{10}\geq 2\sqrt{5}$

 

 

Hãy tìm giá trị lớn nhất của $P=\frac{1}{x^{2}}+\frac{2}{y^{2}}+\frac{3}{z^{2}}$


Bài viết đã được chỉnh sửa nội dung bởi Belphegor Varia: 09-06-2015 - 20:47

$ \textbf{NMQ}$

Wait a minute, You have enough time. Also tomorrow will come 

Just take off her or give me a ride 

Give me one day or one hour or just one minute for a short word 

 


#233
khanghaxuan

khanghaxuan

    Trung úy

  • Thành viên
  • 969 Bài viết

Bài 96 (VMO 2001A ): Xét các số thực dương $x,y,z$ thoả mãn hệ điều kiện sau :

$1$ , $\frac{1}{\sqrt{2}}\leq z< \min (x\sqrt{2},y\sqrt{3})$
$2$ , $x+z\sqrt{3}\geq \sqrt{6}$
$3$ ,  $y\sqrt{3}+z\sqrt{10}\geq 2\sqrt{5}$

 

 

Hãy tìm giá trị lớn nhất của $P=\frac{1}{x^{2}}+\frac{2}{y^{2}}+\frac{3}{z^{2}}$

Hình như phải là  : $\frac{1}{\sqrt{2}}\leq z< \frac{\min (x\sqrt{2},y\sqrt{3})}{2}$ :)) 


Điều tôi muốn biết trước tiên không phải là bạn đã thất bại ra sao mà là bạn đã chấp nhận nó như thế nào .

- A.Lincoln -

#234
longatk08

longatk08

    Sĩ quan

  • Thành viên
  • 350 Bài viết

Spoiler

 

97.(VMO 2006_Bảng B)

 

Tìm số thực $k$ lớn nhất sao cho với mọi $a,b,c$ thực dương thỏa $abc=1$.Ta có BĐT sau:

 

$\frac{1}{a^2}+\frac{1}{b^2}+\frac{1}{c^2}+3k\geq (k+1)(a+b+c)$

 

98.(Romania 2008)

 

Tìm hằng số $k$ lớn nhất để bất đẳng thức sau đúng:

 

$(a+b+c)(\frac{1}{a+b}+\frac{1}{b+c}+\frac{1}{a+c}-k)\geq k$

Trong đó $a,b,c$ thực dương thỏa $a+b+c=ab+bc+ac$

 

99.(Olyimpic toán nữ sinh Trunq Quốc 2007).

Cho $a,b,c$ thực dương thỏa mãn $a+b+c=1$.CMR:

 

$\sqrt{a+\frac{(b-c)^2}{4}}+\sqrt{b}+\sqrt{c}\leq \sqrt{3}$

 

P/s: Dành vinh dự post bài thứ 100 cho người khác  :icon10: !

 

@Khanghaxuan : Đang định để chủ topic (ducvipdh12) đăng bài thứ 100  :icon10: Thấy sao ? 


Bài viết đã được chỉnh sửa nội dung bởi khanghaxuan: 10-06-2015 - 08:39


#235
ducvipdh12

ducvipdh12

    Sĩ quan

  • Thành viên
  • 454 Bài viết

vinh dự quá,câu 100 là do mình đăng :))

Câu 100: ( Kazakhstan NMO 2015 )

Chứng minh rằng:

$\frac{1}{2^2}+\frac{1}{3^2}+...+\frac{1}{(n+1)^2}< n.(1-\frac{1}{\sqrt[n]{2}})$

Khang coi tổng hợp lại được rồi đó em 

@Khanghaxuan: Em đang tổng hợp . Dự kiến chiều nay sẽ post :)) 


Bài viết đã được chỉnh sửa nội dung bởi khanghaxuan: 10-06-2015 - 08:14

FAN THẦY THÔNG,ANH CẨN,THẦY VINH :icon6: :icon6:

#236
Pham Quoc Thang

Pham Quoc Thang

    Trung sĩ

  • Thành viên
  • 160 Bài viết

Bài 99:
$VT^2 \leq 3[(a+\frac{(b-c)^2}{4})+\frac{(\sqrt{b}+\sqrt{c})^2}{4}+\frac{(\sqrt{b}+\sqrt{c})^2}{4}]$
Ta đi chứng minh: $\frac{(b-c)^2}{4}+\frac{(\sqrt{b}+\sqrt{c})^2}{4}+\frac{(\sqrt{b}+\sqrt{c})^2}{4} \leq b+c $
$ \Leftrightarrow (b-c)^2+4\sqrt{bc} \leq 2(b+c) $
Ta có: $(b-c)^2=(\sqrt{b}+\sqrt{c})^2.(\sqrt{b}-\sqrt{c})^2 \leq 2(b+c).(\sqrt{b}-\sqrt{c})^2 \leq 2(\sqrt{b}-\sqrt{c})^2 =2(b+c)-4\sqrt{bc} $
Suy ra :$(b-c)^2+4\sqrt{bc} \leq 2(b+c) $
Vậy ta có:$VT \leq \sqrt{3} $


Bài viết đã được chỉnh sửa nội dung bởi khanghaxuan: 10-06-2015 - 08:35


#237
Hoang Tung 126

Hoang Tung 126

    Thiếu tá

  • Thành viên
  • 2061 Bài viết

Spoiler

 

97.(VMO 2006_Bảng B)

 

Tìm số thực $k$ lớn nhất sao cho với mọi $a,b,c$ thực dương thỏa $abc=1$.Ta có BĐT sau:

 

$\frac{1}{a^2}+\frac{1}{b^2}+\frac{1}{c^2}+3k\geq (k+1)(a+b+c)$

 

 

- Chọn $a=x^2,b=c=\frac{1}{x}$

 

BĐT $< = > \frac{1}{x^4}+\frac{1}{\frac{1}{x^2}}+\frac{1}{\frac{1}{x^2}}+3k\geq (k+1)(x^2+\frac{2}{x})$

$< = > \frac{1}{x^4}+2x^2+3k\geq (k+1)(x^2+\frac{2}{x})< = > \frac{1}{x^4}+2x^2-x^2-\frac{2}{x}\geq k(x^2+\frac{2}{x}-3)$

$< = > \frac{1}{x^4}+x^2-\frac{2}{x}\geq k(x^2+\frac{2}{x}-3)< = > \frac{(x-1)^2(x^2+x+1)^2}{x^4}\geq k(\frac{(x-1)^2(x+2)}{x})$

 

- Với $x\not\equiv 1= > \frac{(x^2+x+1)^2}{x^4}\geq \frac{k(x+2)}{x}= > k\leq \frac{(x^2+x+1)^2}{x^3(x+2)}$

 

    $= > k\leq \lim_{x->+\infty }\frac{(x^2+x+1)^2}{x^3(x+2)}=\lim_{x->+\infty }\frac{(1+\frac{1}{x}+\frac{1}{x^2})^2}{1+\frac{2}{x}}=1$  (Do $\lim_{x->+\infty }\frac{1}{x}=0$)

 

  Từ đó $k\leq 1= > > k_{max}=1$. Ta chứng minh $k=1$ là giá trị lớn nhất thỏa mãn bài toán

 

 Thật vậy ,thay $k=1$ vào BĐT 

 

   $= > \frac{1}{a^2}+\frac{1}{b^2}+\frac{1}{c^2}+3\geq 2(a+b+c)< = > b^2c^2+c^2a^2+a^2b^2+3\geq 2(a+b+c)$  (Do $abc=1$)

 

 Theo BĐT Schur bậc 3 ta có :$a^2b^2+b^2c^2+c^2a^2+3=(\sqrt[3]{a^2b^2})^3+(\sqrt[3]{b^2c^2})^3+(\sqrt[3]{c^2a^2})^3+3\sqrt[3]{(a^2b^2.b^2c^2.c^2a^2)}$

$\geq \sqrt[3]{a^2b^2}.\sqrt[3]{b^2c^2}(\sqrt[3]{a^2b^2}+\sqrt[3]{b^2c^2})+\sqrt[3]{b^2c^2}.\sqrt[3]{c^2a^2}(\sqrt[3]{b^2c^2}+\sqrt[3]{c^2a^2})+\sqrt[3]{c^2a^2}.\sqrt[3]{a^2b^2}(\sqrt[3]{c^2a^2}+\sqrt[3]{a^2b^2})$

$\geq \sqrt[3]{(ab^2c)^2}.2\sqrt[3]{ab^2c}+\sqrt[3]{(abc^2)^2}.2\sqrt[3]{abc^2}+\sqrt[3]{(a^2bc)^2}.2\sqrt[3]{a^2bc}$

$=\sqrt[3]{(abc)^3.b^3}+\sqrt[3]{(abc)^3.c^3}+\sqrt[3]{(abc)^3.a^3}=\sqrt[3]{a^3}+\sqrt[3]{b^3}+\sqrt[3]{c^3}=a+b+c$

 

  $= > a^2b^2+b^2c^2+c^2a^2+3\geq 2(a+b+c)$

 

 (Do áp dụng Cosi và $abc=1$)

 

  Do đó ta có ĐPCM. Dấu = xảy ra khi $a=b=c=1$


Bài viết đã được chỉnh sửa nội dung bởi Hoang Tung 126: 10-06-2015 - 07:21


#238
khanghaxuan

khanghaxuan

    Trung úy

  • Thành viên
  • 969 Bài viết

Ta cũng tìm thấy chặn dưới của bài 99 là : $\sqrt{a+\frac{(b-c)^2}{4}}+\sqrt{b}+\sqrt{c}\geq 1$ :))


Điều tôi muốn biết trước tiên không phải là bạn đã thất bại ra sao mà là bạn đã chấp nhận nó như thế nào .

- A.Lincoln -

#239
Hoang Tung 126

Hoang Tung 126

    Thiếu tá

  • Thành viên
  • 2061 Bài viết

Spoiler

 

98.(Romania 2008)

 

Tìm hằng số $k$ lớn nhất để bất đẳng thức sau đúng:

 

$(a+b+c)(\frac{1}{a+b}+\frac{1}{b+c}+\frac{1}{a+c}-k)\geq k$

Trong đó $a,b,c$ thực dương thỏa $a+b+c=ab+bc+ac$

 

 

BDT $< = > k\leq \frac{(a+b+c)(\frac{1}{a+b}+\frac{1}{b+c}+\frac{1}{c+a})}{1+a+b+c}$

     $= > k\leq max(\frac{(a+b+c)(\frac{1}{a+b}+\frac{1}{b+c}+\frac{1}{c+a})}{1+a+b+c})$ (Do để tìm giá trị lớn nhất của $k$)

 

Ta chứng minh $max(\frac{(a+b+c)(\frac{1}{a+b}+\frac{1}{b+c}+\frac{1}{c+a})}{1+a+b+c})=\frac{9}{8}$

 

   Thật vậy,  BĐT 

 

$< = > 8(a+b+c)(\frac{1}{a+b}+\frac{1}{b+c}+\frac{1}{c+a})\leq 9+\frac{9(a+b+c)^2}{ab+bc+ac}$

  (Do thay $1=\frac{a+b+c}{ab+bc+ac}$)

 

 $< = > 4\left [ (a+b)+(b+c)+(c+a) \right ](\frac{1}{a+b}+\frac{1}{b+c}+\frac{1}{c+a})\leq 9+\frac{9(a+b+c)^2}{ab+bc+ac}$

$< = > 4\left [ \left [ (a+b)+(b+c)+(c+a) \right ](\frac{1}{a+b}+\frac{1}{b+c}+\frac{1}{c+a}) -9\right ]\leq9(\frac{(a+b+c)^2}{ab+bc+ac}-3)$

$< = > 8\left [ \frac{(a-b)^2}{(c+a)(c+b)}+\frac{(b-c)^2}{(b+a)(c+a)}+\frac{(c-a)^2}{(c+b)(a+b)} \right ]\leq\frac{9\left [ (a-b)^2+(b-c)^2+(c-a)^2 \right ]}{ab+bc+ac}$

$< = > (a-b)^2(\frac{9}{ab+bc+ac}-\frac{8}{(a+c)(b+c)})+(b-c)^2(\frac{9}{ab+bc+ac}-\frac{8}{(b+a)(c+a)})+(c-a)^2(\frac{9}{ab+bc+ac}-\frac{8}{(c+b)(a+b)})\geq 0$ (1)

 

 Đặt $S_{a}=\frac{9}{ab+bc+ac}-\frac{8}{(c+a)(b+a)}=\frac{ab+bc+ac+9a^2}{(\sum ab)(c+a)(b+a)}> 0$

       $S_{b}=\frac{9}{ab+bc+ac}-\frac{8}{(b+a)(b+c)}=\frac{ab+bc+ac+9b^2}{(\sum ab)(b+a)(b+c)}> 0$

      $S_{c}=\frac{9}{ab+bc+ac}-\frac{8}{(c+a)(c+b)}=\frac{ab+bc+ac+9c^2}{(\sum ab)(c+a)(c+b)}> 0$

 

Do đó $S_{a}> 0,S_{b}> 0,S_{c}> 0$ nên BĐT (1) đúng và ta có ĐPCM.

 

    Vậy $k_{max}=\frac{9}{8}$ là giá trị cần tìm


Bài viết đã được chỉnh sửa nội dung bởi Hoang Tung 126: 10-06-2015 - 09:16


#240
khanghaxuan

khanghaxuan

    Trung úy

  • Thành viên
  • 969 Bài viết

Vinh dự giải câu thứ 100  **== 

Lời giải câu 100 : 

Ta dùng biến đổi tương đương : 

$\frac{1}{2^{2}}+...+\frac{1}{(n+1)^{2}}<n(1-\frac{1}{\sqrt[n]{2}})$

$\Leftrightarrow (\frac{1}{2^{2}}-1)+...+(\frac{1}{(n+1)^{2}}-1)<\frac{-n}{\sqrt[n]{2}}$

$\Leftrightarrow (1-\frac{1}{2^{2}})+...+(1-\frac{1}{(n+1)^{2}})>\frac{n}{\sqrt[n]{2}}$

$\Leftrightarrow \frac{1(1+2)}{(1+1)^{2}}+...+\frac{n(n+2)}{(n+1)^{2}}>\frac{n}{\sqrt[n]{2}}(*)$

Ta sẽ chứng minh $(*)$ đúng . Thật vậy áp dụng AM-GM ta có : 

$VT(*)>n\sqrt[n]{\frac{2.3^{2}.4^{2}.....n^{2}(n+1)(n+2)}{2^{2}.3^{2}....(n+1)^{2}}}=n\sqrt[n]{\frac{n+2}{2(n+1)}}>n\sqrt[n]{\frac{n+2}{2(n+2)}}=n\sqrt[n]{\frac{1}{2}}$


Điều tôi muốn biết trước tiên không phải là bạn đã thất bại ra sao mà là bạn đã chấp nhận nó như thế nào .

- A.Lincoln -




1 người đang xem chủ đề

0 thành viên, 1 khách, 0 thành viên ẩn danh